There are no ethical choices under first-past-the-post voting. We must instead make a decision that reduces the most harm.

  • null
    link
    fedilink
    arrow-up
    6
    ·
    2 months ago

    Whether you think that applies in this particular case is another question

    If this was what you were presenting this as (a logical response to the argument above) then it shouldn’t be another question. It should apply directly to this argument.

    Your comment only applies to a negotiation between 2 parties and doesn’t address the actual problem at hand whatsoever. So yeah, its not a logical response to the above argument at all.

    • OBJECTION!@lemmy.ml
      link
      fedilink
      arrow-up
      5
      arrow-down
      1
      ·
      2 months ago

      It establishes the logical framework for the opposing case. Making the opposing case requires additional assumptions, such as, where your minimum requirements ought to be set, exactly how good/bad Biden is, etc. Those would be tangents that I don’t really want to get sidetracked by, because my goal was just to establish the logical framework for the opposing case. My comment was long enough as it is, and I’ve frequently had comments that long been (rudely) dismissed as being too long. My purpose for that comment is not to persuade but to explain.

      • null
        link
        fedilink
        arrow-up
        5
        ·
        2 months ago

        It certainly does not establish “the logical framework” for the opposing case. Again, as I explained, the framework deals with 2 parties negotiating, which is not applicable to the argument presented.

        • OBJECTION!@lemmy.ml
          link
          fedilink
          arrow-up
          6
          arrow-down
          1
          ·
          2 months ago

          You haven’t provided any reason why the situations aren’t comparable. If you introduce more parties, it doesn’t change the dynamics of the situation.

          • null
            link
            fedilink
            English
            arrow-up
            5
            ·
            edit-2
            2 months ago

            Because the parties you established are the voter, and the party asking for votes. Those are not the parties presented in the original argument.

            If you introduce more parties, it doesn’t change the dynamics of the situation.

            Of course it does.

            • OBJECTION!@lemmy.ml
              link
              fedilink
              arrow-up
              5
              arrow-down
              1
              ·
              2 months ago

              Because the parties you established are the voter, and the party asking for votes. Those are not the parties presented in the original argument.

              That’s called an analogy.

              Of course it does.

              No it doesn’t.

              • null
                link
                fedilink
                English
                arrow-up
                4
                ·
                2 months ago

                That’s called an analogy.

                Not when it isn’t analogous to the situation presented. Which yours is not.

                No it doesn’t.

                Prove it.

                • OBJECTION!@lemmy.ml
                  link
                  fedilink
                  arrow-up
                  5
                  arrow-down
                  1
                  ·
                  2 months ago

                  Prove it’s not. You’re the one claiming that the distinction makes it not analogous. I don’t know why you think that would change it so it’s impossible for me to address your reasons.

                  • null
                    link
                    fedilink
                    English
                    arrow-up
                    5
                    ·
                    edit-2
                    2 months ago

                    Prove it’s not. You’re the one claiming that the distinction makes it not analogous.

                    That’s not at all how the burden of proof works.

                    I don’t know why you think that would change it so it’s impossible for me to address your reasons.

                    You’re leaping to the assumption that the scenario you provided is even analogous to the one you replied to. It isn’t. You need to start by proving that it is.